If $x_1,x_2,ldots,x_n$ are the roots for $1+x+x^2+ldots+x^n=0$, find the value of...












7












$begingroup$



Let $x_1,x_2,ldots,x_n$ be the roots for $1+x+x^2+ldots+x^n=0$. Find the value of
$$P(1)=frac{1}{x_1-1}+frac{1}{x_2-1}+ldots+frac{1}{x_n-1}$$




Source: IME entrance exam (Military Institute of Engineering, Brazil), date not provided (possibly from the 1950s)



My attempt:
Developing expression $P(1)$, replacing the 1 by $x$, follows
$$P(x)=frac{(x_2-x)cdots (x_n-x)+ldots+(x_1-x)cdots (x_{n-1}-x)}{(x_1-x)(x_2-x)cdots (x_n-x)}$$



As $x_1,x_2,ldots,x_n$ are the roots, it must be true that



$$Q(x)=(x-x_1)cdots(x-x_n)=1+x+x^2+ldots+x^n$$
and
$$Q(1)=(1-x_1)cdots(1-x_n)=n+1$$
Therefore the denominator of $P(1)$ is
$$(-1)^{n} (n+1).$$ But I could not find a way to simplify the numerator.



Another fact that is probably useful is that
$$1+x^{n+1}=(1-x)(x^n+x^{n-1}+ldots+x+1)$$
with roots that are 1 in addition of the given roots $x_1,x_2,ldots,x_n$ for the original equation, that is



$$x_k=text{cis}(frac{2kpi}{n+1}), k=1,ldots,n.$$



This is as far as I could go...



Hints and full answers are welcomed.










share|cite|improve this question











$endgroup$












  • $begingroup$
    Related : math.stackexchange.com/questions/2988339/…
    $endgroup$
    – Arnaud D.
    Dec 17 '18 at 12:07
















7












$begingroup$



Let $x_1,x_2,ldots,x_n$ be the roots for $1+x+x^2+ldots+x^n=0$. Find the value of
$$P(1)=frac{1}{x_1-1}+frac{1}{x_2-1}+ldots+frac{1}{x_n-1}$$




Source: IME entrance exam (Military Institute of Engineering, Brazil), date not provided (possibly from the 1950s)



My attempt:
Developing expression $P(1)$, replacing the 1 by $x$, follows
$$P(x)=frac{(x_2-x)cdots (x_n-x)+ldots+(x_1-x)cdots (x_{n-1}-x)}{(x_1-x)(x_2-x)cdots (x_n-x)}$$



As $x_1,x_2,ldots,x_n$ are the roots, it must be true that



$$Q(x)=(x-x_1)cdots(x-x_n)=1+x+x^2+ldots+x^n$$
and
$$Q(1)=(1-x_1)cdots(1-x_n)=n+1$$
Therefore the denominator of $P(1)$ is
$$(-1)^{n} (n+1).$$ But I could not find a way to simplify the numerator.



Another fact that is probably useful is that
$$1+x^{n+1}=(1-x)(x^n+x^{n-1}+ldots+x+1)$$
with roots that are 1 in addition of the given roots $x_1,x_2,ldots,x_n$ for the original equation, that is



$$x_k=text{cis}(frac{2kpi}{n+1}), k=1,ldots,n.$$



This is as far as I could go...



Hints and full answers are welcomed.










share|cite|improve this question











$endgroup$












  • $begingroup$
    Related : math.stackexchange.com/questions/2988339/…
    $endgroup$
    – Arnaud D.
    Dec 17 '18 at 12:07














7












7








7


2



$begingroup$



Let $x_1,x_2,ldots,x_n$ be the roots for $1+x+x^2+ldots+x^n=0$. Find the value of
$$P(1)=frac{1}{x_1-1}+frac{1}{x_2-1}+ldots+frac{1}{x_n-1}$$




Source: IME entrance exam (Military Institute of Engineering, Brazil), date not provided (possibly from the 1950s)



My attempt:
Developing expression $P(1)$, replacing the 1 by $x$, follows
$$P(x)=frac{(x_2-x)cdots (x_n-x)+ldots+(x_1-x)cdots (x_{n-1}-x)}{(x_1-x)(x_2-x)cdots (x_n-x)}$$



As $x_1,x_2,ldots,x_n$ are the roots, it must be true that



$$Q(x)=(x-x_1)cdots(x-x_n)=1+x+x^2+ldots+x^n$$
and
$$Q(1)=(1-x_1)cdots(1-x_n)=n+1$$
Therefore the denominator of $P(1)$ is
$$(-1)^{n} (n+1).$$ But I could not find a way to simplify the numerator.



Another fact that is probably useful is that
$$1+x^{n+1}=(1-x)(x^n+x^{n-1}+ldots+x+1)$$
with roots that are 1 in addition of the given roots $x_1,x_2,ldots,x_n$ for the original equation, that is



$$x_k=text{cis}(frac{2kpi}{n+1}), k=1,ldots,n.$$



This is as far as I could go...



Hints and full answers are welcomed.










share|cite|improve this question











$endgroup$





Let $x_1,x_2,ldots,x_n$ be the roots for $1+x+x^2+ldots+x^n=0$. Find the value of
$$P(1)=frac{1}{x_1-1}+frac{1}{x_2-1}+ldots+frac{1}{x_n-1}$$




Source: IME entrance exam (Military Institute of Engineering, Brazil), date not provided (possibly from the 1950s)



My attempt:
Developing expression $P(1)$, replacing the 1 by $x$, follows
$$P(x)=frac{(x_2-x)cdots (x_n-x)+ldots+(x_1-x)cdots (x_{n-1}-x)}{(x_1-x)(x_2-x)cdots (x_n-x)}$$



As $x_1,x_2,ldots,x_n$ are the roots, it must be true that



$$Q(x)=(x-x_1)cdots(x-x_n)=1+x+x^2+ldots+x^n$$
and
$$Q(1)=(1-x_1)cdots(1-x_n)=n+1$$
Therefore the denominator of $P(1)$ is
$$(-1)^{n} (n+1).$$ But I could not find a way to simplify the numerator.



Another fact that is probably useful is that
$$1+x^{n+1}=(1-x)(x^n+x^{n-1}+ldots+x+1)$$
with roots that are 1 in addition of the given roots $x_1,x_2,ldots,x_n$ for the original equation, that is



$$x_k=text{cis}(frac{2kpi}{n+1}), k=1,ldots,n.$$



This is as far as I could go...



Hints and full answers are welcomed.







algebra-precalculus complex-numbers contest-math






share|cite|improve this question















share|cite|improve this question













share|cite|improve this question




share|cite|improve this question








edited Dec 14 '18 at 9:17







bluemaster

















asked Dec 13 '18 at 19:23









bluemasterbluemaster

1,653519




1,653519












  • $begingroup$
    Related : math.stackexchange.com/questions/2988339/…
    $endgroup$
    – Arnaud D.
    Dec 17 '18 at 12:07


















  • $begingroup$
    Related : math.stackexchange.com/questions/2988339/…
    $endgroup$
    – Arnaud D.
    Dec 17 '18 at 12:07
















$begingroup$
Related : math.stackexchange.com/questions/2988339/…
$endgroup$
– Arnaud D.
Dec 17 '18 at 12:07




$begingroup$
Related : math.stackexchange.com/questions/2988339/…
$endgroup$
– Arnaud D.
Dec 17 '18 at 12:07










5 Answers
5






active

oldest

votes


















4












$begingroup$

$1$ and $x_1,ldots,x_n$ are the roots of $z^{n+1}-1=0$. They are
the $(n+1)$-th roots of unity.
Then $1/(z^{n+1}-1)$
has a partial fraction expansion of the form
$$frac1{z^{n+1}-1}=frac a{z-1}+sum_{k=1}^nfrac{b_k}{x_kz-1}.$$
Multiplying by $z-1$ and setting $z=1$ gives $a=1/(n+1)$.
Multiplying by $x_kz-1$ and setting $z=1/x_k$ gives $b_k=1/(n+1)$.
Therefore
$$sum_{k=1}^nfrac1{x_kz-1}=frac{n+1}{z^{n+1}-1}-frac1{z-1}.$$
Letting $zto1$ gives
$$sum_{k=1}^nfrac1{x_k-1}=lim_{zto1}frac{n-z-z^2-cdots-z^n}{z^{n+1}-1}
=frac{-1-2-cdots-n}{n+1}=-frac n2.$$



ADDED IN EDIT.



Here's a trick proof. Let $S$ denote the sum in question. As the
reciprocals of the $x_k$ are the $x_k$ again (in a different order) then
$$S=sum_{k=1}^nfrac1{x_k^{-1}-1}=sum_{k=1}^nfrac{x_k}{1-x_k}
$$

and
$$2S=sum_{k=1}^nleft(frac1{x_k-1}+frac{x_k}{1-x_k}right)
=sum_{k=1}^n(-1)=-n.$$






share|cite|improve this answer











$endgroup$





















    4












    $begingroup$

    Applying the same technique as per this answer
    $$Q(x)=prodlimits_{k=1}^n(x-x_k) text{ and }
    Q'(x)=sumlimits_{i=1}^{n}prodlimits_{k=1,kne i}^{n}left(x-x_kright)$$

    then
    $$frac{Q'(x)}{Q(x)}=sumlimits_{i=1}^{n}frac{1}{x-x_i}$$
    or
    $$-frac{Q'(1)}{Q(1)}=sumlimits_{i=1}^{n}frac{1}{x_i-1}$$
    But $Q(1)=n+1$ and $Q'(x)=1+2x+3x^3+...+nx^{n-1}Rightarrow Q'(1)=frac{n(n+1)}{2}$ and
    $$sumlimits_{i=1}^{n}frac{1}{x_i-1}=-frac{n}{2}$$






    share|cite|improve this answer









    $endgroup$





















      3












      $begingroup$

      We have $$x^{n+1}-1=0 (1)$$ with $xne1$



      Set $dfrac1{x-1}=yiff x=dfrac{y+1}y$



      Replace the value of $x$ in terms of $y$ in $(1)$ to form an $n$ degree equation in $y$ $$left(dfrac{y+1}yright)^{n+1}=1$$



      $$impliesbinom{n+1}1 y^n+binom{n+1}2 y^{n-1}+cdots+1=0$$



      Now apply Vieta's formula to find $$sum_{r=1}^ndfrac1{x_r-1}=sum_{r=1}^ny_r=-dfrac{binom{n+1}2}{binom{n+1}1}=?$$






      share|cite|improve this answer











      $endgroup$





















        2












        $begingroup$

        Simply note that $1,x_1,ldots, x_n$ are roots of $$
        x^{n+1} -1 = 0.$$
        Thus we have $0,x_1-1,ldots, x_n-1$ are roots of
        $$
        (x+1)^{n+1} -1 = xsum_{j=0}^{n} binom{n+1}{j+1}x^j = 0.
        $$

        Hence, $z_i = x_i -1$, $1leq i leq n$ are roots of
        $$
        sum_{j=0}^{n} binom{n+1}{j+1}z^j = 0,
        $$
        and by Vieta's formula,
        $$
        sum_{i=1}^n frac{1}{z_i} = frac{sum_{i=1}^n z_1z_2cdots z_{i-1}widehat{z_i}z_{i+1}cdots z_n}{z_1z_2cdots z_{n-1}z_n} = -frac{binom{n+1}{2}}{binom{n+1}{1}} = -frac{n}{2},
        $$
        where $widehat{z_i}$ means the variable is omitted in the calculation.






        share|cite|improve this answer











        $endgroup$





















          1












          $begingroup$

          First, the denominator is, I think, $(-1)^nQ(1)$.
          Here is a hint for the numerator: how can you write $Q’(x)$?






          share|cite|improve this answer









          $endgroup$













            Your Answer





            StackExchange.ifUsing("editor", function () {
            return StackExchange.using("mathjaxEditing", function () {
            StackExchange.MarkdownEditor.creationCallbacks.add(function (editor, postfix) {
            StackExchange.mathjaxEditing.prepareWmdForMathJax(editor, postfix, [["$", "$"], ["\\(","\\)"]]);
            });
            });
            }, "mathjax-editing");

            StackExchange.ready(function() {
            var channelOptions = {
            tags: "".split(" "),
            id: "69"
            };
            initTagRenderer("".split(" "), "".split(" "), channelOptions);

            StackExchange.using("externalEditor", function() {
            // Have to fire editor after snippets, if snippets enabled
            if (StackExchange.settings.snippets.snippetsEnabled) {
            StackExchange.using("snippets", function() {
            createEditor();
            });
            }
            else {
            createEditor();
            }
            });

            function createEditor() {
            StackExchange.prepareEditor({
            heartbeatType: 'answer',
            autoActivateHeartbeat: false,
            convertImagesToLinks: true,
            noModals: true,
            showLowRepImageUploadWarning: true,
            reputationToPostImages: 10,
            bindNavPrevention: true,
            postfix: "",
            imageUploader: {
            brandingHtml: "Powered by u003ca class="icon-imgur-white" href="https://imgur.com/"u003eu003c/au003e",
            contentPolicyHtml: "User contributions licensed under u003ca href="https://creativecommons.org/licenses/by-sa/3.0/"u003ecc by-sa 3.0 with attribution requiredu003c/au003e u003ca href="https://stackoverflow.com/legal/content-policy"u003e(content policy)u003c/au003e",
            allowUrls: true
            },
            noCode: true, onDemand: true,
            discardSelector: ".discard-answer"
            ,immediatelyShowMarkdownHelp:true
            });


            }
            });














            draft saved

            draft discarded


















            StackExchange.ready(
            function () {
            StackExchange.openid.initPostLogin('.new-post-login', 'https%3a%2f%2fmath.stackexchange.com%2fquestions%2f3038472%2fif-x-1-x-2-ldots-x-n-are-the-roots-for-1xx2-ldotsxn-0-find-the-value%23new-answer', 'question_page');
            }
            );

            Post as a guest















            Required, but never shown

























            5 Answers
            5






            active

            oldest

            votes








            5 Answers
            5






            active

            oldest

            votes









            active

            oldest

            votes






            active

            oldest

            votes









            4












            $begingroup$

            $1$ and $x_1,ldots,x_n$ are the roots of $z^{n+1}-1=0$. They are
            the $(n+1)$-th roots of unity.
            Then $1/(z^{n+1}-1)$
            has a partial fraction expansion of the form
            $$frac1{z^{n+1}-1}=frac a{z-1}+sum_{k=1}^nfrac{b_k}{x_kz-1}.$$
            Multiplying by $z-1$ and setting $z=1$ gives $a=1/(n+1)$.
            Multiplying by $x_kz-1$ and setting $z=1/x_k$ gives $b_k=1/(n+1)$.
            Therefore
            $$sum_{k=1}^nfrac1{x_kz-1}=frac{n+1}{z^{n+1}-1}-frac1{z-1}.$$
            Letting $zto1$ gives
            $$sum_{k=1}^nfrac1{x_k-1}=lim_{zto1}frac{n-z-z^2-cdots-z^n}{z^{n+1}-1}
            =frac{-1-2-cdots-n}{n+1}=-frac n2.$$



            ADDED IN EDIT.



            Here's a trick proof. Let $S$ denote the sum in question. As the
            reciprocals of the $x_k$ are the $x_k$ again (in a different order) then
            $$S=sum_{k=1}^nfrac1{x_k^{-1}-1}=sum_{k=1}^nfrac{x_k}{1-x_k}
            $$

            and
            $$2S=sum_{k=1}^nleft(frac1{x_k-1}+frac{x_k}{1-x_k}right)
            =sum_{k=1}^n(-1)=-n.$$






            share|cite|improve this answer











            $endgroup$


















              4












              $begingroup$

              $1$ and $x_1,ldots,x_n$ are the roots of $z^{n+1}-1=0$. They are
              the $(n+1)$-th roots of unity.
              Then $1/(z^{n+1}-1)$
              has a partial fraction expansion of the form
              $$frac1{z^{n+1}-1}=frac a{z-1}+sum_{k=1}^nfrac{b_k}{x_kz-1}.$$
              Multiplying by $z-1$ and setting $z=1$ gives $a=1/(n+1)$.
              Multiplying by $x_kz-1$ and setting $z=1/x_k$ gives $b_k=1/(n+1)$.
              Therefore
              $$sum_{k=1}^nfrac1{x_kz-1}=frac{n+1}{z^{n+1}-1}-frac1{z-1}.$$
              Letting $zto1$ gives
              $$sum_{k=1}^nfrac1{x_k-1}=lim_{zto1}frac{n-z-z^2-cdots-z^n}{z^{n+1}-1}
              =frac{-1-2-cdots-n}{n+1}=-frac n2.$$



              ADDED IN EDIT.



              Here's a trick proof. Let $S$ denote the sum in question. As the
              reciprocals of the $x_k$ are the $x_k$ again (in a different order) then
              $$S=sum_{k=1}^nfrac1{x_k^{-1}-1}=sum_{k=1}^nfrac{x_k}{1-x_k}
              $$

              and
              $$2S=sum_{k=1}^nleft(frac1{x_k-1}+frac{x_k}{1-x_k}right)
              =sum_{k=1}^n(-1)=-n.$$






              share|cite|improve this answer











              $endgroup$
















                4












                4








                4





                $begingroup$

                $1$ and $x_1,ldots,x_n$ are the roots of $z^{n+1}-1=0$. They are
                the $(n+1)$-th roots of unity.
                Then $1/(z^{n+1}-1)$
                has a partial fraction expansion of the form
                $$frac1{z^{n+1}-1}=frac a{z-1}+sum_{k=1}^nfrac{b_k}{x_kz-1}.$$
                Multiplying by $z-1$ and setting $z=1$ gives $a=1/(n+1)$.
                Multiplying by $x_kz-1$ and setting $z=1/x_k$ gives $b_k=1/(n+1)$.
                Therefore
                $$sum_{k=1}^nfrac1{x_kz-1}=frac{n+1}{z^{n+1}-1}-frac1{z-1}.$$
                Letting $zto1$ gives
                $$sum_{k=1}^nfrac1{x_k-1}=lim_{zto1}frac{n-z-z^2-cdots-z^n}{z^{n+1}-1}
                =frac{-1-2-cdots-n}{n+1}=-frac n2.$$



                ADDED IN EDIT.



                Here's a trick proof. Let $S$ denote the sum in question. As the
                reciprocals of the $x_k$ are the $x_k$ again (in a different order) then
                $$S=sum_{k=1}^nfrac1{x_k^{-1}-1}=sum_{k=1}^nfrac{x_k}{1-x_k}
                $$

                and
                $$2S=sum_{k=1}^nleft(frac1{x_k-1}+frac{x_k}{1-x_k}right)
                =sum_{k=1}^n(-1)=-n.$$






                share|cite|improve this answer











                $endgroup$



                $1$ and $x_1,ldots,x_n$ are the roots of $z^{n+1}-1=0$. They are
                the $(n+1)$-th roots of unity.
                Then $1/(z^{n+1}-1)$
                has a partial fraction expansion of the form
                $$frac1{z^{n+1}-1}=frac a{z-1}+sum_{k=1}^nfrac{b_k}{x_kz-1}.$$
                Multiplying by $z-1$ and setting $z=1$ gives $a=1/(n+1)$.
                Multiplying by $x_kz-1$ and setting $z=1/x_k$ gives $b_k=1/(n+1)$.
                Therefore
                $$sum_{k=1}^nfrac1{x_kz-1}=frac{n+1}{z^{n+1}-1}-frac1{z-1}.$$
                Letting $zto1$ gives
                $$sum_{k=1}^nfrac1{x_k-1}=lim_{zto1}frac{n-z-z^2-cdots-z^n}{z^{n+1}-1}
                =frac{-1-2-cdots-n}{n+1}=-frac n2.$$



                ADDED IN EDIT.



                Here's a trick proof. Let $S$ denote the sum in question. As the
                reciprocals of the $x_k$ are the $x_k$ again (in a different order) then
                $$S=sum_{k=1}^nfrac1{x_k^{-1}-1}=sum_{k=1}^nfrac{x_k}{1-x_k}
                $$

                and
                $$2S=sum_{k=1}^nleft(frac1{x_k-1}+frac{x_k}{1-x_k}right)
                =sum_{k=1}^n(-1)=-n.$$







                share|cite|improve this answer














                share|cite|improve this answer



                share|cite|improve this answer








                edited Dec 13 '18 at 20:55

























                answered Dec 13 '18 at 19:37









                Lord Shark the UnknownLord Shark the Unknown

                105k1160133




                105k1160133























                    4












                    $begingroup$

                    Applying the same technique as per this answer
                    $$Q(x)=prodlimits_{k=1}^n(x-x_k) text{ and }
                    Q'(x)=sumlimits_{i=1}^{n}prodlimits_{k=1,kne i}^{n}left(x-x_kright)$$

                    then
                    $$frac{Q'(x)}{Q(x)}=sumlimits_{i=1}^{n}frac{1}{x-x_i}$$
                    or
                    $$-frac{Q'(1)}{Q(1)}=sumlimits_{i=1}^{n}frac{1}{x_i-1}$$
                    But $Q(1)=n+1$ and $Q'(x)=1+2x+3x^3+...+nx^{n-1}Rightarrow Q'(1)=frac{n(n+1)}{2}$ and
                    $$sumlimits_{i=1}^{n}frac{1}{x_i-1}=-frac{n}{2}$$






                    share|cite|improve this answer









                    $endgroup$


















                      4












                      $begingroup$

                      Applying the same technique as per this answer
                      $$Q(x)=prodlimits_{k=1}^n(x-x_k) text{ and }
                      Q'(x)=sumlimits_{i=1}^{n}prodlimits_{k=1,kne i}^{n}left(x-x_kright)$$

                      then
                      $$frac{Q'(x)}{Q(x)}=sumlimits_{i=1}^{n}frac{1}{x-x_i}$$
                      or
                      $$-frac{Q'(1)}{Q(1)}=sumlimits_{i=1}^{n}frac{1}{x_i-1}$$
                      But $Q(1)=n+1$ and $Q'(x)=1+2x+3x^3+...+nx^{n-1}Rightarrow Q'(1)=frac{n(n+1)}{2}$ and
                      $$sumlimits_{i=1}^{n}frac{1}{x_i-1}=-frac{n}{2}$$






                      share|cite|improve this answer









                      $endgroup$
















                        4












                        4








                        4





                        $begingroup$

                        Applying the same technique as per this answer
                        $$Q(x)=prodlimits_{k=1}^n(x-x_k) text{ and }
                        Q'(x)=sumlimits_{i=1}^{n}prodlimits_{k=1,kne i}^{n}left(x-x_kright)$$

                        then
                        $$frac{Q'(x)}{Q(x)}=sumlimits_{i=1}^{n}frac{1}{x-x_i}$$
                        or
                        $$-frac{Q'(1)}{Q(1)}=sumlimits_{i=1}^{n}frac{1}{x_i-1}$$
                        But $Q(1)=n+1$ and $Q'(x)=1+2x+3x^3+...+nx^{n-1}Rightarrow Q'(1)=frac{n(n+1)}{2}$ and
                        $$sumlimits_{i=1}^{n}frac{1}{x_i-1}=-frac{n}{2}$$






                        share|cite|improve this answer









                        $endgroup$



                        Applying the same technique as per this answer
                        $$Q(x)=prodlimits_{k=1}^n(x-x_k) text{ and }
                        Q'(x)=sumlimits_{i=1}^{n}prodlimits_{k=1,kne i}^{n}left(x-x_kright)$$

                        then
                        $$frac{Q'(x)}{Q(x)}=sumlimits_{i=1}^{n}frac{1}{x-x_i}$$
                        or
                        $$-frac{Q'(1)}{Q(1)}=sumlimits_{i=1}^{n}frac{1}{x_i-1}$$
                        But $Q(1)=n+1$ and $Q'(x)=1+2x+3x^3+...+nx^{n-1}Rightarrow Q'(1)=frac{n(n+1)}{2}$ and
                        $$sumlimits_{i=1}^{n}frac{1}{x_i-1}=-frac{n}{2}$$







                        share|cite|improve this answer












                        share|cite|improve this answer



                        share|cite|improve this answer










                        answered Dec 13 '18 at 21:08









                        rtybasertybase

                        11.1k21533




                        11.1k21533























                            3












                            $begingroup$

                            We have $$x^{n+1}-1=0 (1)$$ with $xne1$



                            Set $dfrac1{x-1}=yiff x=dfrac{y+1}y$



                            Replace the value of $x$ in terms of $y$ in $(1)$ to form an $n$ degree equation in $y$ $$left(dfrac{y+1}yright)^{n+1}=1$$



                            $$impliesbinom{n+1}1 y^n+binom{n+1}2 y^{n-1}+cdots+1=0$$



                            Now apply Vieta's formula to find $$sum_{r=1}^ndfrac1{x_r-1}=sum_{r=1}^ny_r=-dfrac{binom{n+1}2}{binom{n+1}1}=?$$






                            share|cite|improve this answer











                            $endgroup$


















                              3












                              $begingroup$

                              We have $$x^{n+1}-1=0 (1)$$ with $xne1$



                              Set $dfrac1{x-1}=yiff x=dfrac{y+1}y$



                              Replace the value of $x$ in terms of $y$ in $(1)$ to form an $n$ degree equation in $y$ $$left(dfrac{y+1}yright)^{n+1}=1$$



                              $$impliesbinom{n+1}1 y^n+binom{n+1}2 y^{n-1}+cdots+1=0$$



                              Now apply Vieta's formula to find $$sum_{r=1}^ndfrac1{x_r-1}=sum_{r=1}^ny_r=-dfrac{binom{n+1}2}{binom{n+1}1}=?$$






                              share|cite|improve this answer











                              $endgroup$
















                                3












                                3








                                3





                                $begingroup$

                                We have $$x^{n+1}-1=0 (1)$$ with $xne1$



                                Set $dfrac1{x-1}=yiff x=dfrac{y+1}y$



                                Replace the value of $x$ in terms of $y$ in $(1)$ to form an $n$ degree equation in $y$ $$left(dfrac{y+1}yright)^{n+1}=1$$



                                $$impliesbinom{n+1}1 y^n+binom{n+1}2 y^{n-1}+cdots+1=0$$



                                Now apply Vieta's formula to find $$sum_{r=1}^ndfrac1{x_r-1}=sum_{r=1}^ny_r=-dfrac{binom{n+1}2}{binom{n+1}1}=?$$






                                share|cite|improve this answer











                                $endgroup$



                                We have $$x^{n+1}-1=0 (1)$$ with $xne1$



                                Set $dfrac1{x-1}=yiff x=dfrac{y+1}y$



                                Replace the value of $x$ in terms of $y$ in $(1)$ to form an $n$ degree equation in $y$ $$left(dfrac{y+1}yright)^{n+1}=1$$



                                $$impliesbinom{n+1}1 y^n+binom{n+1}2 y^{n-1}+cdots+1=0$$



                                Now apply Vieta's formula to find $$sum_{r=1}^ndfrac1{x_r-1}=sum_{r=1}^ny_r=-dfrac{binom{n+1}2}{binom{n+1}1}=?$$







                                share|cite|improve this answer














                                share|cite|improve this answer



                                share|cite|improve this answer








                                edited Dec 14 '18 at 4:41

























                                answered Dec 13 '18 at 19:29









                                lab bhattacharjeelab bhattacharjee

                                226k15157275




                                226k15157275























                                    2












                                    $begingroup$

                                    Simply note that $1,x_1,ldots, x_n$ are roots of $$
                                    x^{n+1} -1 = 0.$$
                                    Thus we have $0,x_1-1,ldots, x_n-1$ are roots of
                                    $$
                                    (x+1)^{n+1} -1 = xsum_{j=0}^{n} binom{n+1}{j+1}x^j = 0.
                                    $$

                                    Hence, $z_i = x_i -1$, $1leq i leq n$ are roots of
                                    $$
                                    sum_{j=0}^{n} binom{n+1}{j+1}z^j = 0,
                                    $$
                                    and by Vieta's formula,
                                    $$
                                    sum_{i=1}^n frac{1}{z_i} = frac{sum_{i=1}^n z_1z_2cdots z_{i-1}widehat{z_i}z_{i+1}cdots z_n}{z_1z_2cdots z_{n-1}z_n} = -frac{binom{n+1}{2}}{binom{n+1}{1}} = -frac{n}{2},
                                    $$
                                    where $widehat{z_i}$ means the variable is omitted in the calculation.






                                    share|cite|improve this answer











                                    $endgroup$


















                                      2












                                      $begingroup$

                                      Simply note that $1,x_1,ldots, x_n$ are roots of $$
                                      x^{n+1} -1 = 0.$$
                                      Thus we have $0,x_1-1,ldots, x_n-1$ are roots of
                                      $$
                                      (x+1)^{n+1} -1 = xsum_{j=0}^{n} binom{n+1}{j+1}x^j = 0.
                                      $$

                                      Hence, $z_i = x_i -1$, $1leq i leq n$ are roots of
                                      $$
                                      sum_{j=0}^{n} binom{n+1}{j+1}z^j = 0,
                                      $$
                                      and by Vieta's formula,
                                      $$
                                      sum_{i=1}^n frac{1}{z_i} = frac{sum_{i=1}^n z_1z_2cdots z_{i-1}widehat{z_i}z_{i+1}cdots z_n}{z_1z_2cdots z_{n-1}z_n} = -frac{binom{n+1}{2}}{binom{n+1}{1}} = -frac{n}{2},
                                      $$
                                      where $widehat{z_i}$ means the variable is omitted in the calculation.






                                      share|cite|improve this answer











                                      $endgroup$
















                                        2












                                        2








                                        2





                                        $begingroup$

                                        Simply note that $1,x_1,ldots, x_n$ are roots of $$
                                        x^{n+1} -1 = 0.$$
                                        Thus we have $0,x_1-1,ldots, x_n-1$ are roots of
                                        $$
                                        (x+1)^{n+1} -1 = xsum_{j=0}^{n} binom{n+1}{j+1}x^j = 0.
                                        $$

                                        Hence, $z_i = x_i -1$, $1leq i leq n$ are roots of
                                        $$
                                        sum_{j=0}^{n} binom{n+1}{j+1}z^j = 0,
                                        $$
                                        and by Vieta's formula,
                                        $$
                                        sum_{i=1}^n frac{1}{z_i} = frac{sum_{i=1}^n z_1z_2cdots z_{i-1}widehat{z_i}z_{i+1}cdots z_n}{z_1z_2cdots z_{n-1}z_n} = -frac{binom{n+1}{2}}{binom{n+1}{1}} = -frac{n}{2},
                                        $$
                                        where $widehat{z_i}$ means the variable is omitted in the calculation.






                                        share|cite|improve this answer











                                        $endgroup$



                                        Simply note that $1,x_1,ldots, x_n$ are roots of $$
                                        x^{n+1} -1 = 0.$$
                                        Thus we have $0,x_1-1,ldots, x_n-1$ are roots of
                                        $$
                                        (x+1)^{n+1} -1 = xsum_{j=0}^{n} binom{n+1}{j+1}x^j = 0.
                                        $$

                                        Hence, $z_i = x_i -1$, $1leq i leq n$ are roots of
                                        $$
                                        sum_{j=0}^{n} binom{n+1}{j+1}z^j = 0,
                                        $$
                                        and by Vieta's formula,
                                        $$
                                        sum_{i=1}^n frac{1}{z_i} = frac{sum_{i=1}^n z_1z_2cdots z_{i-1}widehat{z_i}z_{i+1}cdots z_n}{z_1z_2cdots z_{n-1}z_n} = -frac{binom{n+1}{2}}{binom{n+1}{1}} = -frac{n}{2},
                                        $$
                                        where $widehat{z_i}$ means the variable is omitted in the calculation.







                                        share|cite|improve this answer














                                        share|cite|improve this answer



                                        share|cite|improve this answer








                                        edited Dec 14 '18 at 6:13

























                                        answered Dec 14 '18 at 5:52









                                        SongSong

                                        15.9k1739




                                        15.9k1739























                                            1












                                            $begingroup$

                                            First, the denominator is, I think, $(-1)^nQ(1)$.
                                            Here is a hint for the numerator: how can you write $Q’(x)$?






                                            share|cite|improve this answer









                                            $endgroup$


















                                              1












                                              $begingroup$

                                              First, the denominator is, I think, $(-1)^nQ(1)$.
                                              Here is a hint for the numerator: how can you write $Q’(x)$?






                                              share|cite|improve this answer









                                              $endgroup$
















                                                1












                                                1








                                                1





                                                $begingroup$

                                                First, the denominator is, I think, $(-1)^nQ(1)$.
                                                Here is a hint for the numerator: how can you write $Q’(x)$?






                                                share|cite|improve this answer









                                                $endgroup$



                                                First, the denominator is, I think, $(-1)^nQ(1)$.
                                                Here is a hint for the numerator: how can you write $Q’(x)$?







                                                share|cite|improve this answer












                                                share|cite|improve this answer



                                                share|cite|improve this answer










                                                answered Dec 13 '18 at 19:31









                                                MindlackMindlack

                                                4,780210




                                                4,780210






























                                                    draft saved

                                                    draft discarded




















































                                                    Thanks for contributing an answer to Mathematics Stack Exchange!


                                                    • Please be sure to answer the question. Provide details and share your research!

                                                    But avoid



                                                    • Asking for help, clarification, or responding to other answers.

                                                    • Making statements based on opinion; back them up with references or personal experience.


                                                    Use MathJax to format equations. MathJax reference.


                                                    To learn more, see our tips on writing great answers.




                                                    draft saved


                                                    draft discarded














                                                    StackExchange.ready(
                                                    function () {
                                                    StackExchange.openid.initPostLogin('.new-post-login', 'https%3a%2f%2fmath.stackexchange.com%2fquestions%2f3038472%2fif-x-1-x-2-ldots-x-n-are-the-roots-for-1xx2-ldotsxn-0-find-the-value%23new-answer', 'question_page');
                                                    }
                                                    );

                                                    Post as a guest















                                                    Required, but never shown





















































                                                    Required, but never shown














                                                    Required, but never shown












                                                    Required, but never shown







                                                    Required, but never shown

































                                                    Required, but never shown














                                                    Required, but never shown












                                                    Required, but never shown







                                                    Required, but never shown







                                                    Popular posts from this blog

                                                    Bundesstraße 106

                                                    Le Mesnil-Réaume

                                                    Ida-Boy-Ed-Garten